Foros-FIUBA Foros HostingPortal
 FAQ  •  Buscar  •  Wiki  •  Apuntes  •  Planet  •  Mapa  •  Eyeon  •  Chat
Preferencias  •  Grupos de Usuarios
Registrarse  •  Perfil  •  Entrá para ver tus mensajes privados  •  Login
Ver tema siguiente
Ver tema anterior

Responder al tema Ver tema anteriorEnviar por mail a un amigo.Mostrar una Lista de los Usuarios que vieron este TemaGuardar este Tema como un archivoPrintable versionEntrá para ver tus mensajes privadosVer tema siguiente
Autor Mensaje
martin_ush
Nivel 2


Edad: 36
Registrado: 21 Jul 2010
Mensajes: 10


argentina.gif
MensajePublicado: Vie Jul 23, 2010 6:48 pm  Asunto:  (Sin Asunto) Responder citandoFin de la PáginaVolver arriba

deep.seba escribió:
che martin y como lo planteaste vos? xq lo que yo hice fue calcular lo que serian los minimos de difraccion por fraunhoffer y propagar errores para ver si de alguna manera podia llegar a que sea menor al 10 %

[tex]b*\sin \theta  = n\lambda  [/tex]

aproximamos el sen por la tangente, siendo "y" la distancia al primer minimo, "d" la distancia a la pantalla y "b" el diametro de la rendija.
[tex] b*\frac{y}{d} = n\lambda [/tex]
[tex] b = \frac{{\lambda d}}{y}[/tex]

aca propagamos errores
[tex]\Delta b = \frac{\lambda }{y}\Delta d + \frac{\lambda }{{y^2 }}\Delta y [/tex]

y este es el resultado con la incerteza maxima posible que tendria que dar (menor al 10%)
[tex] b = 0.1mm \pm 0.01mm[/tex]
[tex]\Delta b \le 0.01mm [/tex]

pero como hago para corroborar que efectivamente esta dentro de ese rango la incerteza? no tengo ni la distancia con la pantalla ni la distancia del primer minimo.

saludos


Libra Género:Masculino Gato OfflineGalería Personal de martin_ushVer perfil de usuarioEnviar mensaje privado
martin_ush
Nivel 2


Edad: 36
Registrado: 21 Jul 2010
Mensajes: 10


argentina.gif
MensajePublicado: Vie Jul 23, 2010 6:49 pm  Asunto:  (Sin Asunto) Responder citandoFin de la PáginaVolver arriba

deep.seba escribió:
che martin y como lo planteaste vos? xq lo que yo hice fue calcular lo que serian los minimos de difraccion por fraunhoffer y propagar errores para ver si de alguna manera podia llegar a que sea menor al 10 %

[tex]b*\sin \theta  = n\lambda  [/tex]

aproximamos el sen por la tangente, siendo "y" la distancia al primer minimo, "d" la distancia a la pantalla y "b" el diametro de la rendija.
[tex] b*\frac{y}{d} = n\lambda [/tex]
[tex] b = \frac{{\lambda d}}{y}[/tex]

aca propagamos errores
[tex]\Delta b = \frac{\lambda }{y}\Delta d + \frac{\lambda }{{y^2 }}\Delta y [/tex]

y este es el resultado con la incerteza maxima posible que tendria que dar (menor al 10%)
[tex] b = 0.1mm \pm 0.01mm[/tex]
[tex]\Delta b \le 0.01mm [/tex]

pero como hago para corroborar que efectivamente esta dentro de ese rango la incerteza? no tengo ni la distancia con la pantalla ni la distancia del primer minimo.

saludos


Libra Género:Masculino Gato OfflineGalería Personal de martin_ushVer perfil de usuarioEnviar mensaje privado
martin_ush
Nivel 2


Edad: 36
Registrado: 21 Jul 2010
Mensajes: 10


argentina.gif
MensajePublicado: Vie Jul 23, 2010 6:51 pm  Asunto:  (Sin Asunto) Responder citandoFin de la PáginaVolver arriba

esta bien lo q hiciste. yo hice mas o menos lo mismo. la distancia D la elegis vos convenientemente.para la distanticia Y, cuando hagas la difraccion, se van a ver los maximos. a vos te interesa el max central. va a tener un ancho. donde muere el ancho es donde empieza el minimo. entonces 2Y es igual a ese ancho con n= 1.

espero haber sido claro. saludos

pd: me dieron la nota hoy y aprobé, estoy saltando en una pata. jaja

pd2: perdon, no se que hice, queria citar un mensaje y me salio cualquier cosa.


Libra Género:Masculino Gato OfflineGalería Personal de martin_ushVer perfil de usuarioEnviar mensaje privado
Yankey
Nivel 5


Edad: 33
Registrado: 02 Abr 2010
Mensajes: 181

Carrera: Electricista
argentina.gif
MensajePublicado: Sab Jul 24, 2010 1:03 am  Asunto:  (Sin Asunto) Responder citandoFin de la PáginaVolver arriba

Bistek escribió:
Posteo mi resolución del 1.a de ese del wiki a ver si alguien me dice donde me equivoco porque no llego a esos resultados (omitiendo varias justificaciones para no hacerlo tan denso)

A) Primero planteo que durante el choque se conserva la cantidad de movimiento

[tex]\Delta p = 0[/tex]
[tex]p_o=p_f[/tex]
[tex]m_b \ V_{bo} + m_B \ V_{Bo} = (m_b + m_B) \ V_f[/tex]
donde el subindice b es para los datos de la bala y el subíndice B para el bloque, de acá despejo la velocidad final y me da:
[tex]V_f = \frac{0.1 kg \cdot 200m/s}{0.1 kg + 2kg} = 9.52 m/s [/tex]

Entonces la bala y el bloque van con esta velocidad constante hasta el inicio de la rampa, porque en ese trayecto de 2m no hay rozamiento

Lo próximo que hago es calcular por energía la velocidad del bloque justo antes de tocar el resorte, tengo que
[tex]W_{FNC} = \Delta E_{MEC} [/tex]

Calculo [tex]W_{FNC}[/tex] que corresponde solo a la fuerza de rozamiento porq la normal no hace trabajo
La fuerza de rozamiento es, tomando el sistema de ejes inclinado clasico
[tex]F_{roz} = \mu \cdot N = \mu \cdot P_y[/tex]
[tex]F_{roz} = \mu \cdot cos(30) \cdot P[/tex]
[tex]F_{roz} = 0.1 \cdot 0.866 \cdot 20.58N [/tex]
[tex]F_{roz} = 1.782N[/tex]

Entonces, como recorre 6m y es opuesta al movimiento, [tex]W_{FNC} =1.782N \cdot 6m = -10,69J[/tex]

Volviendo al planteo de antes
[tex]W_{FNC} = \Delta E_{MEC} [/tex]
[tex]W_{FNC} = E_{MECf} - E_{MECo}[/tex]
[tex]-10.69J = m \cdot g \cdot h_f + \frac{m \cdot V_f^2}{2} - \frac{m \cdot V_o^2}{2}[/tex]
[tex]-10.69J = 2.1kg \cdot 9.8 \cdot 3m + \frac{2.1kg \cdot V_f^2}{2} - \frac{2.1kg \cdot (9.52m/2)^2}{2}[/tex]

Donde saque la altura por trigonometría, 3 metros, y la velocidad inicial es la que saqué antes. Entonces despejo la velocidad final y me da
[tex]V_f = 4.65m/s [/tex]

El último paso lo que hago es calcular la compresión del resorte en este tramo que se conserva la energia
[tex]\Delta E_{MEC} = 0[/tex]
[tex]E_{MECo} = E_{MECf}[/tex]
[tex]\frac{m \cdot V_o^2}{2} = m \cdot g \cdot h_f + \frac{k \cdot (\Delta x)^2}{2}[/tex]
La altura final no la tengo entonces la pongo en función de la compresión del resorte
[tex]h_f = sen(30) \cdot \Delta x[/tex]

Entonces me queda una cuadrática de donde sacaria la compresión:
[tex] \frac{k \cdot (\Delta x)^2}{2} + m \cdot g \cdot sen(30) \cdot  \Delta x - \frac{m \cdot V_o^2}{2} = 0[/tex]

y de aca me dan dos valores posibles donde deberia desechar el que no tenga valor físico, pero la cuenta me da como 10 veces menos al resultado que pusieron ahí

Que estoy haciendo mal?

No hice este ejercicio, asi que no se si el resultado que obtenes es bvueno o malo.
Pero la altura final hf es en realidad
[tex]h_f = sen(30) \cdot (\Delta x + 6)[/tex] si tu sistema de referencia es 0 a la altura en la cual se encontraba inicialmente en reposo el bloque.


Tauro Género:Masculino Cabra OfflineGalería Personal de YankeyVer perfil de usuarioEnviar mensaje privado
Snajdan
Nivel 5



Registrado: 21 Oct 2009
Mensajes: 191
Ubicación: Banfield.
Carrera: Química
russia.gif
MensajePublicado: Sab Jul 24, 2010 12:36 pm  Asunto:  (Sin Asunto) Responder citandoFin de la PáginaVolver arriba

El 4 a) cómo se resuelve?

_________________
SNAJ.

 Género:Masculino  OfflineGalería Personal de SnajdanVer perfil de usuarioEnviar mensaje privadoMSN Messenger
thew0rldismin3
Nivel 7



Registrado: 18 Dic 2008
Mensajes: 437

Carrera: No especificada
argentina.gif
MensajePublicado: Sab Jul 24, 2010 1:12 pm  Asunto:  (Sin Asunto) Responder citandoFin de la PáginaVolver arriba

Image

Si hay algún error avisen. Me resulta medio raro el resultado que me dio para los focos.
Saludos


 Género:Masculino  OcultoGalería Personal de thew0rldismin3Ver perfil de usuarioEnviar mensaje privado
Bistek
Nivel 8



Registrado: 07 May 2010
Mensajes: 691

Carrera: Informática
CARRERA.informatica.3.jpg
MensajePublicado: Sab Jul 24, 2010 1:47 pm  Asunto:  (Sin Asunto) Responder citandoFin de la PáginaVolver arriba

el 4a (del que subieron al wiki q pusieron en la pagina 1) lo hice así:

Tengo entendido que hay varias convenciones para los signos, la que yo uso es tomar el eje positivo EN CONTRA de los rayos de la luz que ingresan a la lente.
Bueno a vos te dan el tamaño del objeto (2m) y el tamaño de la imagen (3m), por lo tanto, en la "formulita" del aumento

[tex]Aumento = \frac{h'}{h} = \frac{s'}{s}[/tex]
[tex]\frac{3m}{2m} = \frac{s'}{s}  \mbox {   de aca sale que }  s'=1.5s[/tex]

Si ves el dibujo te das cuenta que como la imagen no esta invertida, entonces el aumento es positivo. Entonces, s' y s, que son las distancias a la imagen y objeto respectivamente, tienen el mismo signo. Esto lo que te esta diciendo es que tanto el objeto como la imagen estan del mismo lado de la lente.
Por lo tanto, o bien son ambos positivos, o bien ambos negativos, pero el enunciado dice que el objeto es real, por lo tanto está del lado de donde proviene la luz. O sea que s y s' son ambos positivos.

Hasta ahí estamos, pero necesito los valores de s y s' para sacar la posición del foco con la formulita de lentes.
Si uso el dato que la distancia entre s y s' es 0.2m, entonces
[tex]s'-s=0.2m[/tex]
[tex]1.5s-s=0.2m[/tex]
[tex]0.5s=0.2m[/tex]
[tex]s=0.4m \ \Rightarrow s'=0.6m[/tex]

Tengo lo que necesito y calculo la distancia focal
[tex]\frac{1}{s} - \frac{1}{s'} = \frac{1}{f}[/tex]
[tex]\frac{1}{0.4m} - \frac{1}{0.6m} = \frac{1}{f}[/tex]
[tex]f=0.833m[/tex] Como el foco es positivo, entonces la lente es convergente.

Los resultados dicen que la lente esta a 0.4m a la IZQUIERDA del objeto (porque usé que s' es mas grande que s, del otro modo me quedaban cantidades negativas para las distancias y se contradice con lo deducido antes)


 Género:Masculino  OcultoGalería Personal de BistekVer perfil de usuarioEnviar mensaje privado
Snajdan
Nivel 5



Registrado: 21 Oct 2009
Mensajes: 191
Ubicación: Banfield.
Carrera: Química
russia.gif
MensajePublicado: Sab Jul 24, 2010 2:16 pm  Asunto:  (Sin Asunto) Responder citandoFin de la PáginaVolver arriba

Gracias por contestar, ya entendi.

Y en el 4 a2) esta bien si digo que puede ser bicóncava, debido a que esto depende en el medio en el cual se encuentre la lente?

_________________
SNAJ.

 Género:Masculino  OfflineGalería Personal de SnajdanVer perfil de usuarioEnviar mensaje privadoMSN Messenger
juanc@/1989
Nivel 5


Edad: 34
Registrado: 21 Dic 2009
Mensajes: 171
Ubicación: Lanus Este
Carrera: Civil
argentina.gif
MensajePublicado: Dom Jul 25, 2010 12:01 pm  Asunto:  (Sin Asunto) Responder citandoFin de la PáginaVolver arriba

Buenas.
Tenia un par de dudas con respecto al ejercicio 3

en el 3b) lamda 1= 2L= 2m , f1= 10hz, T= 20N y m= 2kg lamda 2=L=1m, f2=20hz, T=80N y m= 8kg

hasta ahi todo bien creo...

ahora en el item 3c que piden las ondas viajeras



2pi f1= w1=20pi k1= 2pi/lamda 1= pi

Entonces: y(x,t)=Asen(pi x - 20pi t) se propaga hacia la derecha)
y(x,t)=Asen(pix + 20pi t) se propaga hacia la izquierda)

luego t=3/2 T t=3/2. 0.1= 0.15 s

Luego me piden la onda para t=0.15s

y(x, 0.15)= Asen(pi x - 3pi)= Asen(pi x - pi)
y(x,0.15) = Asen(pi x + 3pi)= Asen(pi x + pi)


Esta bien??

en el armonico

w2= 2pi.f2= 40pi k2= 2pi/lamda2= 2pi

y(x,t)= Asen(2pi x - 40pi t) propaga a la derecha

y(x,t)= Asen(2pi x + 40pi t) propaga a la izquierda

t= 0.15s

y(x,0.15)= Asen(2pi x - 6pi)= Asen(2pi x)

y(x,0.15)= Asen(2pi x - 6pi)= Asen(2pi x)

esta bien??


osea mi duda es en considerar la fase inicial fi cero

y(x,t)= Asen (kx - wt + fi) donde fi= fase inicial del movimiento

como se calcula fi?? puedo considerar fi= cero, es decir como las ondas

se generan en el punto central de la figura, desde ese punto tomo como

origen del movimiento de las ondas??

Perdon por no usar latex

_________________
"La ignorancia, es la enfermedad que al mundo apesta"

Leo Género:Masculino Serpiente OfflineGalería Personal de juanc@/1989Ver perfil de usuarioEnviar mensaje privadoMSN Messenger
Trigger
Nivel 8



Registrado: 06 Ago 2008
Mensajes: 524

Carrera: Industrial
argentina.gif
MensajePublicado: Dom Jul 25, 2010 9:35 pm  Asunto:  (Sin Asunto) Responder citandoFin de la PáginaVolver arriba

Bistek, en el parcial que postearon en el 4a, la imagen mide 2 y el objeto 3


 Género:Masculino  OfflineGalería Personal de TriggerVer perfil de usuarioEnviar mensaje privado
Bistek
Nivel 8



Registrado: 07 May 2010
Mensajes: 691

Carrera: Informática
CARRERA.informatica.3.jpg
MensajePublicado: Lun Jul 26, 2010 12:12 pm  Asunto:  (Sin Asunto) Responder citandoFin de la PáginaVolver arriba

Oh, pequeño detalle, q boludo, igual supongo que se captó la idea de resolución


 Género:Masculino  OcultoGalería Personal de BistekVer perfil de usuarioEnviar mensaje privado
thew0rldismin3
Nivel 7



Registrado: 18 Dic 2008
Mensajes: 437

Carrera: No especificada
argentina.gif
MensajePublicado: Lun Jul 26, 2010 3:32 pm  Asunto:  (Sin Asunto) Responder citandoFin de la PáginaVolver arriba

Hola gente! estoy intentando hacer el 2, pero usando una "separación" del cuerpo rígido, en este caso el cilindro que gira, en movimientos de traslación del centro de masa y rotación alrededor del cm de la siguiente forma:

Image

Nose si alguien entenderá algo de ese planteo que hice, pero en nuestro curso nos enseñaron a encarar los problemas de Cuerpo rígido de esta forma :S separando el problema en traslación y rotación y usando cuplas de fuerzas.
El problema viene en la fuerza de vínculo Fv del cilindro.
Si planteo que tiene la misma dirección que el Peso, tendrá el mismo ángulo de 45º con la vertical, entonces sucede lo que puse en rojo.
Ahora si planteo que el ángulo en realidad no es el mismo, ya que la dirección de Fv no coincide con la del Peso del cilindro, entonces me aparecería como nueva incógnica ese angulo alfa...
Nose si se entiende :/

¿Alguien sabe cómo hacerlo de esta forma? ¿O si eso está bien?

Estuve leyendo lo que ya discutieron sobre ese problema, pero me gustaría saber si lo que hice está medianamente bien, ya que además es así como me lo enseñaron y tengo nulas posibilidades de aprenderme otra forma de hacerlo en menos de 1 día :P

Desde ya gracias!
Saludos


 Género:Masculino  OcultoGalería Personal de thew0rldismin3Ver perfil de usuarioEnviar mensaje privado
Trigger
Nivel 8



Registrado: 06 Ago 2008
Mensajes: 524

Carrera: Industrial
argentina.gif
MensajePublicado: Lun Jul 26, 2010 4:22 pm  Asunto:  (Sin Asunto) Responder citandoFin de la PáginaVolver arriba

juanc@/1989: la frecuencia la sacaste haciendo 1/T(periodo) y despues para sacar la Vprop, que formula de frecuencia usaste y porque, f=n.Vprop/2L o f=(2n-1)Vprop/4L


 Género:Masculino  OfflineGalería Personal de TriggerVer perfil de usuarioEnviar mensaje privado
juanc@/1989
Nivel 5


Edad: 34
Registrado: 21 Dic 2009
Mensajes: 171
Ubicación: Lanus Este
Carrera: Civil
argentina.gif
MensajePublicado: Lun Jul 26, 2010 5:00 pm  Asunto:  (Sin Asunto) Responder citandoFin de la PáginaVolver arriba

Trigger escribió:
juanc@/1989: la frecuencia la sacaste haciendo 1/T(periodo) y despues para sacar la Vprop, que formula de frecuencia usaste y porque, f=n.Vprop/2L o f=(2n-1)Vprop/4L



si mira, el tema es el siguiente

como pedian la fundamental, use: f1= 1/2L (T/m)al cuadrado

y luego para el armonico siguiente: 2f1= f2

de todas maneras, los resultados estan bien, lo que tenia dudas del problema 3, es con la fase inicial fi cero, que si la tengo que considerar o no, por lo demas, todo lo que hice en el ejercicio, tiene sentido


Leo Género:Masculino Serpiente OfflineGalería Personal de juanc@/1989Ver perfil de usuarioEnviar mensaje privadoMSN Messenger
Andrês!
Nivel 4


Edad: 33
Registrado: 15 Ago 2009
Mensajes: 110

Carrera: Industrial
argentina.gif
MensajePublicado: Lun Jul 26, 2010 8:14 pm  Asunto:  (Sin Asunto) Responder citandoFin de la PáginaVolver arriba

Subí el tema 2 al wiki

_________________
à bientôt

Escorpio Género:Masculino Caballo OfflineGalería Personal de Andrês!Ver perfil de usuarioEnviar mensaje privado
Mostrar mensajes de anteriores:      
Responder al tema Ver tema anteriorEnviar por mail a un amigo.Mostrar una Lista de los Usuarios que vieron este TemaGuardar este Tema como un archivoPrintable versionEntrá para ver tus mensajes privadosVer tema siguiente

Ver tema siguiente
Ver tema anterior
Podés publicar nuevos temas en este foro
No podés responder a temas en este foro
No podés editar tus mensajes en este foro
No podés borrar tus mensajes en este foro
No podés votar en encuestas en este foro
No Podéspostear archivos en este foro
No Podés bajar archivos de este foro


Todas las horas son ART, ARST (GMT - 3, GMT - 2 Horas)
Protected by CBACK CrackerTracker
365 Attacks blocked.

Powered by phpBB2 Plus, phpBB Styles and Kostenloses Forum based on phpBB © 2001/6 phpBB Group :: FI Theme :: Mods y Créditos

Foros-FIUBA está hosteado en Neolo.com Cloud Hosting

[ Tiempo: 0.5365s ][ Pedidos: 20 (0.3961s) ]